Multivariable Calculus, 7th Edition

Published by Brooks Cole
ISBN 10: 0-53849-787-4
ISBN 13: 978-0-53849-787-9

Chapter 11 - Infinite Sequences and Series - 11.11 Exercises - Page 798: 10

Answer

$$T_{3}(x) =\frac{\pi}{4}+\frac{1}{2}(x-1)-\frac{1}{4}(x-1)^{2}+\frac{1}{12}(x-1)^{3} $$

Work Step by Step

Given $$f(x) =\tan^{-1}x,\ \ a=1$$ Since \begin{align*} f(x) &=\tan^{-1}x\ \ \ \ \ \ \ \ \ \ f(1) =\frac{\pi}{4}\\ f'(x) &=\frac{ 1}{1+x^2}\ \ \ \ \ \ \ f'(1 )=\frac{1}{2}\\ f''(x) &=\frac{-2x}{(1+x^2)^2}\ \ \ \ \ \ \ f''(1 )=\frac{-1}{2}\\ f'''(x) &=\frac{6x^2-2}{(1+x^2)^2}\ \ \ \ \ \ \ f'''(1 )=\frac{1}{2} \end{align*} Then \begin{aligned} T_{3}(x) &=\sum_{n=0}^{3} \frac{f^{(n)}(1)}{n !}(x-1)^{n}\\ &=\frac{\pi}{4}+\frac{1 / 2}{1}(x-1)^{1}+\frac{-1 / 2}{2}(x-1)^{2}+\frac{1 / 2}{6}(x-1)^{3} \\ &=\frac{\pi}{4}+\frac{1}{2}(x-1)-\frac{1}{4}(x-1)^{2}+\frac{1}{12}(x-1)^{3} \end{aligned}
Update this answer!

You can help us out by revising, improving and updating this answer.

Update this answer

After you claim an answer you’ll have 24 hours to send in a draft. An editor will review the submission and either publish your submission or provide feedback.